Quantcast
  • Register
PhysicsOverflow is a next-generation academic platform for physicists and astronomers, including a community peer review system and a postgraduate-level discussion forum analogous to MathOverflow.

Welcome to PhysicsOverflow! PhysicsOverflow is an open platform for community peer review and graduate-level Physics discussion.

Please help promote PhysicsOverflow ads elsewhere if you like it.

News

PO is now at the Physics Department of Bielefeld University!

New printer friendly PO pages!

Migration to Bielefeld University was successful!

Please vote for this year's PhysicsOverflow ads!

Please do help out in categorising submissions. Submit a paper to PhysicsOverflow!

... see more

Tools for paper authors

Submit paper
Claim Paper Authorship

Tools for SE users

Search User
Reclaim SE Account
Request Account Merger
Nativise imported posts
Claim post (deleted users)
Import SE post

Users whose questions have been imported from Physics Stack Exchange, Theoretical Physics Stack Exchange, or any other Stack Exchange site are kindly requested to reclaim their account and not to register as a new user.

Public \(\beta\) tools

Report a bug with a feature
Request a new functionality
404 page design
Send feedback

Attributions

(propose a free ad)

Site Statistics

205 submissions , 163 unreviewed
5,047 questions , 2,200 unanswered
5,345 answers , 22,709 comments
1,470 users with positive rep
816 active unimported users
More ...

  Construct an SO(3) rotation inside the two SU(2) fundamental rotations

+ 5 like - 0 dislike
952 views

We know that two SU(2) fundamentals have multiplication decompositions, such that $$ 2 \otimes 2= 1 \oplus 3.$$ In particular, the 3 has a vector representation of SO(3). While the 1 is the trivial representation of SU(2).

I hope to see the precise SO(3) rotation from the two SU(2) fundamental rotations.

-

So let us first write two SU(2) fundamental objects in terms of an SO(3) object. In particular, we can consider the following three:

$$ |1,1\rangle= \begin{pmatrix} 1\\ 0 \end{pmatrix}\begin{pmatrix} 1\\ 0 \end{pmatrix}= | \uparrow \uparrow \rangle,$$ $$|1,0\rangle ={1 \over \sqrt{2} } (\begin{pmatrix} 1\\ 0 \end{pmatrix} \begin{pmatrix} 0\\ 1 \end{pmatrix} + \begin{pmatrix} 0\\ 1 \end{pmatrix} \begin{pmatrix} 1\\ 0 \end{pmatrix})={1 \over \sqrt{2} }(| \uparrow \downarrow \rangle+ | \downarrow \uparrow \rangle) ,$$ $$|1,-1\rangle = \begin{pmatrix} 0\\ 1 \end{pmatrix}\begin{pmatrix} 0\\ 1 \end{pmatrix}= | \downarrow \downarrow \rangle. $$

where the $| \uparrow \rangle$ and $ \downarrow \rangle$ are in SU(2) fundamentals. And we shothand $| \uparrow \uparrow \rangle \equiv | \uparrow \rangle |\uparrow \rangle $ and so on.

question: How do we rotate between $|1,1\rangle$, $|1,0\rangle$, $|1,-1\rangle$, via two SU(2) rotations acting on two SU(2) fundamentals? Namely, that is, construct an SO(3) rotation inside the two SU(2) fundamental rotations? The SU(2) has three generators, parametrized by $m_x,m_y,m_z$; how do we write down the generic SO(3) rotations from two SU(2) rotations?

Let us consider an example, an SU(2) rotation $U$ acting on the SU(2) fundamental $\begin{pmatrix} 1\\ 0 \end{pmatrix}$ give rise to $$ U \begin{pmatrix} 1\\ 0 \end{pmatrix}= \begin{pmatrix} \cos(\frac{\theta}{2})+{i m_z} \sin(\frac{\theta}{2}) & (i m_x -m_y) \sin(\frac{\theta}{2}) \\ (i m_x +m_y) \sin(\frac{\theta}{2}) & \cos(\frac{\theta}{2})-{i m_z} \sin(\frac{\theta}{2}) \\ \end{pmatrix} \begin{pmatrix} 1\\ 0 \end{pmatrix}= \begin{pmatrix} \cos(\frac{\theta}{2})+{i m_z} \sin(\frac{\theta}{2})\\ (i m_x +m_y) \sin(\frac{\theta}{2}) \end{pmatrix} \equiv\cos(\frac{\theta}{2})+{i m_z} \sin(\frac{\theta}{2}) \begin{pmatrix} 1\\ 0 \end{pmatrix} + (i m_x +m_y) \sin(\frac{\theta}{2}) \begin{pmatrix} 0\\ 1 \end{pmatrix} $$

In other words, the SU(2) rotation $U$ (with the $|\vec{m}|^2=1$) rotates SU(2) fundamentals. Two SU(2) rotations act as $$ UU|1,1\rangle = U \begin{pmatrix} 1\\ 0 \end{pmatrix}U \begin{pmatrix} 1\\ 0 \end{pmatrix} = \begin{pmatrix} \cos(\frac{\theta}{2})+{i m_z} \sin(\frac{\theta}{2})\\ (i m_x +m_y) \sin(\frac{\theta}{2}) \end{pmatrix}\begin{pmatrix} \cos(\frac{\theta}{2})+{i m_z} \sin(\frac{\theta}{2})\\ (i m_x +m_y) \sin(\frac{\theta}{2}) \end{pmatrix} $$

Hint: Naively, we like to construct $$ L_{\pm} =L_{x} \pm i L_y, $$ such that $L_{\pm}$ is an operator out of two SU(2) rotations acting on two SU(2) fundamentals, such that it raises/lowers between $|1,1\rangle$, $|1,0\rangle$, $|1,-1\rangle$.

question 2: Is it plausible that two SU(2) are impossible to perform such SO(3) rotations, but we need two U(2) rotations?

This post imported from StackExchange Physics at 2020-11-06 18:48 (UTC), posted by SE-user annie marie heart
asked Apr 21, 2018 in Mathematics by annie marie heart (1,205 points) [ no revision ]

2 Answers

+ 3 like - 0 dislike

The following solution originates from the theory of geometric quantization. I'll not explain the full theory behind it, but I'll give here the solution, then briefly discuss how to check that this is the required solution.

A general $SU(2)$ group element in the fundamental representation can be written as: $$ g = \begin{bmatrix} \alpha & \beta\\ -\bar{\beta}& \bar{\alpha} \end{bmatrix} $$ with $$|\alpha|^2+\beta|^2=1$$ The three dimensional Hilbert space of the three dimensional representation can be parametrized by: $$ \psi (z) = a + b z + cz^2 \quad (1) $$

where $x$ is an indeterminate

The action of $SU(2)$ on this vector space is given by:

$$ (g\cdot \psi)(z) = (-\bar{\beta} z + \bar{\alpha})^2 \psi( \frac{\alpha z + \beta}{-\bar{\beta} z + \bar{\alpha}}) \quad (2)$$

  1. To see that this is a representation, one can check that the composition of the action of two group elements coincides with the action of their product.
  2. To see that this is a faithful $SO(3)$ representation but not a faithful $SU(2)$, we can easily see that for the nontrivial element of the center: $$ g_c = \begin{bmatrix} -1 & 0\\ 0 & -1 \end{bmatrix} $$ We have for every $\psi$ $$ (g_c\cdot \psi)(z) = \psi(z)$$
  3. Although, the "spherical" components $a, b, c$ are complex. To see that the representation is real, one can see that the "Cartesian" components $ (a+c), b, i^{-1}(a-c)$ transform by means of only real combinations of $\alpha$ and $\beta$.
This post imported from StackExchange Physics at 2020-11-06 18:48 (UTC), posted by SE-user David Bar Moshe
answered Apr 22, 2018 by David Bar Moshe (4,355 points) [ no revision ]
@ David Bar Moshe, thanks for the answer, +1. Your answer is in a more advanced level. :)

This post imported from StackExchange Physics at 2020-11-06 18:48 (UTC), posted by SE-user annie marie heart
Then I guess maybe you can also solve this or at least share your opinions: math.stackexchange.com/q/2745276 - thanks.

This post imported from StackExchange Physics at 2020-11-06 18:48 (UTC), posted by SE-user annie marie heart
@ David, it is easy to see from $$ UU|1,1\rangle = U \begin{pmatrix} 1\\ 0 \end{pmatrix}U \begin{pmatrix} 1\\ 0 \end{pmatrix} = \begin{pmatrix} \cos(\frac{\theta}{2})+{i m_z} \sin(\frac{\theta}{2})\\ (i m_x +m_y) \sin(\frac{\theta}{2}) \end{pmatrix}\begin{pmatrix} \cos(\frac{\theta}{2})+{i m_z} \sin(\frac{\theta}{2})\\ (i m_x +m_y) \sin(\frac{\theta}{2}) \end{pmatrix} $$ when $m_y=\pm 1, m_x=m_z=1$ and $\theta=\pi$, we have $$ UU|1,1\rangle = \begin{pmatrix} 0\\ 1 \end{pmatrix} \begin{pmatrix} 0\\ 1 \end{pmatrix}=|1,-1\rangle$$

This post imported from StackExchange Physics at 2020-11-06 18:48 (UTC), posted by SE-user annie marie heart
Do you have any precise answer how to find $U$, such that $$ UU|1,1\rangle = \frac{1}{\sqrt 2} \begin{pmatrix} 1\\ 0 \end{pmatrix}\begin{pmatrix} 0\\ 1 \end{pmatrix} + \frac{1}{\sqrt 2} \begin{pmatrix} 1\\ 0 \end{pmatrix}\begin{pmatrix} 0\\ 1 \end{pmatrix}=|1,0\rangle ?$$

This post imported from StackExchange Physics at 2020-11-06 18:48 (UTC), posted by SE-user annie marie heart
@annie heart there is no such a $U$. Please observe that if you act by some $U \otimes U$ on the highest weight vector: $ \begin{pmatrix} 1\\ 0 \end{pmatrix} \otimes \begin{pmatrix} 1\\ 0 \end{pmatrix}$, then the result will always be a separable vector of the form $ \begin{pmatrix} a\\ b\end{pmatrix} \otimes \begin{pmatrix} a\\ b \end{pmatrix}$. But the vector that you want to reach is entangled, and there is no way to change the entanglement state by a local transformation.

This post imported from StackExchange Physics at 2020-11-06 18:48 (UTC), posted by SE-user David Bar Moshe
Indeed, the entanglement structure is different, then how about construct the rotations out of two U(2) rotations, instead of two SU(2)?

This post imported from StackExchange Physics at 2020-11-06 18:48 (UTC), posted by SE-user annie marie heart
No, $U(2)$ will not change the entanglement structure either, An $SU(3)$ element acting on the three dimensional vector space will be needed.

This post imported from StackExchange Physics at 2020-11-06 18:48 (UTC), posted by SE-user David Bar Moshe
+ 2 like - 0 dislike

Perhaps the following is helpful:

  1. OP's eq. (1) is to be understood as a relation between complex representations of $SU(2)$, i.e complex vector spaces. Recalling that the fundamental $SU(2)$ representation ${\bf 2}\cong \overline{\bf 2}$ is isomorphic to the complex conjugate representation, let us instead consider the isomorphism $$ {\bf 2}\otimes \overline{\bf 2}~\cong~{\bf 1}\oplus {\bf 3}. \tag{A}$$

  2. The left-hand side of eq. (A) can be realized as the real vector space $u(2)$ of $2\times 2$ Hermitian matrices. The group $SU(2)$ acts on $u(2)$ via conjugation. Given a spinor $| \psi\rangle\in {\bf 2}$, then $$ {\bf 1}\oplus su(2)~\cong~ u(2)~\ni~| \psi\rangle \langle\psi | ~=~\frac{1}{2}\sum_{\mu=0}^3x^{\mu} \sigma_{\mu}, \qquad (x^0,x^1,x^2,x^3) ~\in~\mathbb{R}^4. \tag{B}$$ The triplet ${\bf 3}$ corresponds to the traceless part, that is: $su(2)$. Hence the spinor $| \psi\rangle$ represents the 3-vector $\vec{r}=(x^1,x^2,x^3)$. See also this related Phys.SE post.

This post imported from StackExchange Physics at 2020-11-06 18:48 (UTC), posted by SE-user Qmechanic
answered Apr 22, 2018 by Qmechanic (3,120 points) [ no revision ]
Thanks! +1, the same question as to David, do you have a precise rotation from how to find U, such that UU|1,1⟩=(1/√2) (10)(01)+(1/√2)√(10)(01)=|1,0⟩ by some U?

This post imported from StackExchange Physics at 2020-11-06 18:48 (UTC), posted by SE-user annie marie heart

Your answer

Please use answers only to (at least partly) answer questions. To comment, discuss, or ask for clarification, leave a comment instead.
To mask links under text, please type your text, highlight it, and click the "link" button. You can then enter your link URL.
Please consult the FAQ for as to how to format your post.
This is the answer box; if you want to write a comment instead, please use the 'add comment' button.
Live preview (may slow down editor)   Preview
Your name to display (optional):
Privacy: Your email address will only be used for sending these notifications.
Anti-spam verification:
If you are a human please identify the position of the character covered by the symbol $\varnothing$ in the following word:
p$\hbar\varnothing$sicsOverflow
Then drag the red bullet below over the corresponding character of our banner. When you drop it there, the bullet changes to green (on slow internet connections after a few seconds).
Please complete the anti-spam verification




user contributions licensed under cc by-sa 3.0 with attribution required

Your rights
...